Вы находитесь на странице: 1из 14

EE 571 Linear Systems Home Work 1 Page 1 / 14

EE 571 Linear Systems Spring 2014


Home Work # 1


Submitted By: Akhtar Rasool
Student ID: 17197



Faculty of Engineering and Natural Sciences,
Sabanci University, Istanbul
Turkey





EE 571 Linear Systems Home Work 1 Page 2 / 14



Contents Page No.
Question # 01 3
Question # 02 4
Question # 03 5
Question # 04 6
Question # 05 8
Question # 06 8
Question # 07 9
Question # 08 10
Question # 09 13
Question # 10 14















EE 571 Linear Systems Home Work 1 Page 3 / 14

Question # 01
Consider the electrical circuit below. The input voltage source is F an assume the output is the current
|, which is measured across the inductor.

(a) Using the Kirchhoffs Law, derive a differential equation for the circuit above.
(b) By defining state variables as F
c
and |, derive a state-space model.
(c) Determine the transfer function of the circuit above.
Solution (a): Applying the Kirchhoffs Voltages Law gives the following two equations:
F + F
c
+|R
2
+ L
d|
dt
= =
d|
dt
=
R
2
L
|
F
C
L
+
F
L
(1)
| = |
c
+|
R
1
= | = C
dF
C
dt
+
F
C
R
1
=
dF
C
dt
=
1
C
|
1
R
1
C
F
C
(2)
Solution (b): Introducing the state variables as under;
x
1
= | = x
1
=
d|
dt
and x
2
= F
c
= x
2
=
dF
C
dt

So the equations (1) and (2) renders to the following form:
x
1
(t) =
R
2
L
x
1
(t)
1
L
x
2
(t) +
1
L
F(t) (3)
x
2
(t) =
1
C
x
1
(t)
1
R
1
C
x
2
(t) (4)
Hence the state-space model is obtained to be;
_
x
1
(t)
x
2
(t)
_ =

R
2
I

1
I
1
C

1
R
1
C

_
x
1
(t)
x
2
(t)
_ + _
1
I
u
_ I(t)
y = i = x
1
(t) = |1 u] _
x
1
(t)
x
2
(t)
_ + |u]. I(t)
Or
EE 571 Linear Systems Home Work 1 Page 4 / 14

A =

R
2
L

1
L
1
C

1
R
1
C

, B = _
1
L

_ , C = |1 ] onJ D = |]


Solution (c): The transfer function can be determined by using (x) = {C(xI A)
-1
B+D]F(x)
(xI A) = _
x +R
2
L 1L
1C x +1R
1
C
_
Adj(xI A) = _
x +1R
1
C 1L
1C x +R
2
L
_ and |xI A| = _x +
R
2
L
] _x +
1
R
1
C
] +1LC
Y(x)
F(x)
=
I(x)
F(x)
= {C(xI A)
-1
B+D]
Substituting all of the matrices and values in the above, we get as follows;
=
I(x)
F(x)
= |1 ] _
_
x +1R
1
C 1L
1C x + R
2
L
_
[x +
R
2
L
[x +
1
R
1
C
+
1
LC
__
1
L
u
_ +u
=
I(x)
F(x)
= |1 ] _
1
[x +
R
2
L
[x +
1
R
1
C
+
1
LC
__
(x + 1R
1
C)
1
L
1
LC
_
=
I(x)
F(x)
= _
(x + 1R
1
C)
1
L
[x +
R
2
L
[x +
1
R
1
C
+
1
LC
_
Which is the required Transfer function of the given circuit.
Question # 02
Express x = j
4
5
[ as a linear combination of y
1
= j
1
3
[ and y
2
= j
2
2
[.
Solution:
We can write in the following form and will try to determine the values for o
1
onJ o
2
if exist;
x = o
1
y
1
+ o
2
y
2

= j
4
S
[ = o
1
j
1
S
[ +o
2
j
2
2
[
_
4 = o
1
+ 2o
2
(1)
S = So
1
+ 2o
2
(2)
_
Subtractiong eq (2) from eq (1), evolves that 1 = 2o
1
= o
1
= 12,
Substituting the value of o
1
in eq (1) gives o
2
= 74
EE 571 Linear Systems Home Work 1 Page 5 / 14

So,
x = _
1
2
7
4
_
This x represents the linear combination of y
1
and y
2
.
Question # 03
Given the basis __
1

_ , _

1
1
_ , _

__ ur R
3
, which vectors can be removed and replaced by the vector _
4
3
3
_,
while still maintaining a basis? Illustrate this geometrically.
Solution:
a) Lets replace _
4
3
3
_ in the 1st place and explore the independence to conclude basis of the new set.
__
4
3
3
_ , _

1
1
_ , _

__
We can write in the linear combination form as;
_
4o
1
= u
So
1
+ o
2
+o
3
= u
So
1
+o
2
= u
_
Since o
1
= o
2
= o
3
= u so the system is linearly independent. Hence replacing 1st vector of the system
with the given vector does not change the basis.
b) Lets replace _
4
3
3
_ in the 2nd place and explore the independence to conclude basis of the new set.
__
1

_ , _
4
3
3
_ , _

__
We can write in the linear combination form as;
_
o
1
+ 4o
2
= u
So
2
+o
3
= u
So
2
= u
_
Since o
1
= o
2
= o
3
= u so the system is linearly independent. Hence replacing 2nd vector of the system
with the given vector does not change the basis.
c) Lets replace _
4
3
3
_ in the 3rd place and explore the independence to conclude basis of the new set.
EE 571 Linear Systems Home Work 1 Page 6 / 14

__
1

_ , _

1
1
_ , _
4
3
3
__
We can write in the linear combination form as;
_
o
1
+ 4o
3
= u
o
2
+ So
3
= u
o
2
+ So
3
= u
_
Here the last two algebraic equations are identical so we will suppose o
3
= k from which we can deduce
o
2
= Sk & o
1
= 4k which shows that the system is linearly dependent so the replacing 3rd vector of
the given system with the given vector changes the basis.
Geometrically linearly dependent and linearly independent vectors in R
3
are shown as under;

Question # 04
Which of the following sets of vectors form a basis for R
3
?
A = __
3

2
_ , _
7

9
_ , _
4
1
2
__ B = __
1
1

_ , _
3

1
_ , _
5
2
1
__ C = __
1
5
7
_ , _
4

_ , _
1

__
Solution:
(a) For A, we can write the linear form as under;
o
1
_
S
u
2
_ +o
2
_
7
u
9
_ +o
3
_
4
1
2
_ = _
u
u
u
_
_
S 7 4
u u 1
2 9 2
u
u
u
_
The Reduced Echelon form can be determined as follows;
EE 571 Linear Systems Home Work 1 Page 7 / 14

_
1 2 2
u 1S 2
u u 1
u
u
u
_ ,
1
S
R
1
, R
3
2R
1
=

1 u
22
1S
u 1
2
1S
u u 1
u
u
u

,
1
1S
R
2
, R
1
+2R
2


= _
1 u u
u 1 u
u u 1
u
u
u
_ , R
1
+
22
1S
R
3
, R
2
+
2
1S
R
3

Since o
1
= o
2
= o
3
= u; which means that the system is linearly independent so the given system
has a basis for R
3
.
(b) For B, we can write the linear form as under;
o
1
_
1
1
u
_ +o
2
_
S
u
1
_ +o
3
_
S
2
1
_ = _
u
u
u
_
_
1 S S
1 u 2
u 1 1
u
u
u
_
The Reduced Echelon form can be determined as follows;
_
1 S S
u S S
u 1 1
u
u
u
_ , R
2
R
1
= _
1 u 2
u 1 1
u u u
u
u
u
_ ,
1
S
R
2
, R
1
SR
2
, R
3
R
2

Here o
1
= o
2
= u; but o
3
= rcc clcmcnt so let o
3
= k.
Since there is at least one element (o
3
= k , say), a non-zero so the system is linearly dependent which
means the given system does not has basis for R
3
.
(c) For C, we can write the linear form as under;
o
1
_
1
S
7
_ +o
2
_
4
u
6
_ +o
3
_
1
u
u
_ = _
u
u
u
_
_
1 4 1
S u u
7 6 u
u
u
u
_
The Reduced Echelon form is determined as follows;
_
1 4 1
u 2u S
u 22 7
u
u
u
_ , R
2
SR
1
, R
3
7R
1

=

1 u u
u 1
1
4
u u
S
2
u
u
u

,
1
2u
R
2
, R
1
4R
2
, R
3
+22R
2

EE 571 Linear Systems Home Work 1 Page 8 / 14

= _
1 u u
u 1 u
u u 1
u
u
u
_ ,
2
S
R
3
, R
2

1
4
R
3

Since o
1
= o
2
= o
3
= u; so the system is linearly independent so the given system has a basis for R
3
.
Question # 05
Let A
nn
have determinant |A|. Show that for any u H , |uA| = u
n
|A|.
Solution-Proof: Let matrix A of order n by n as follows;
A = _
o
11
o
12
o
21
o
22

o
1n
o
2n

o
n1
o
n2 o
nn
_
Now
|uA| = _
oo
11
oo
12
oo
21
oo
22

oo
1n
oo
2n

o
o
n1
oo
n2 oo
nn
_ = o _
o
11
o
12
oo
21
oo
22

o
1n
oo
2n

o
o
n1
oo
n2 oo
nn
_
Note o is taken as common from R
1
.
= _
oo
11
oo
12
oo
21
oo
22

oo
1n
oo
2n

o
o
n1
oo
n2 oo
nn
_ = o
2
_
o
11
o
12
o
21
o
22

o
1n
o
2n

o
o
n1
oo
n2 oo
nn
_
Note o is taken as common from R
2
.
Similarly by taking o as common from all rows till R
n
produces the following form of the determinant;
= _
oo
11
oo
12
oo
21
oo
22

oo
1n
oo
2n

o
o
n1
oo
n2 oo
nn
_ = o
n
_
o
11
o
12
o
21
o
22

o
1n
o
2n

o
n1
o
n2 o
nn
_ = o
n
|A|
Hence
|uA| = u
n
|A|
Question # 06
Let x and y be n hy 1 column vectors. Show that x
T
y = tr(xy
T
).
Solution:
Let x = _
o
1
o
2

o
n
_ onJ y = _
b
1
b
2

b
n
_ = x
1
= |
o
1
o
2 o
n
] onJ y
1
= |b
1
b
2
b
n
]
The LHS evolves the result,
EE 571 Linear Systems Home Work 1 Page 9 / 14

= x
1
y = |
o
1
o
2 o
n
] _
b
1
b
2

b
n
_ = |o
1
b
1
+ o
2
b
2
+o
n
b
n
] (1)
Now the RHS can be evaluated as under;
RHS= tr(xy
1
) = tr __
o
1
o
2

o
n
_ |b
1
b
2
b
n
]_ = tr _
o
1
b
1
o
1
b
2
o
1
b
n
o
2
b
1
o
2
b
2

o
2
b
n

o
n
b
1
o
n
b
2
o
n
b
n
_
= tr(xy
1
) = |o
1
b
1
+ o
2
b
2
+o
n
b
n
] (2)
From (1) and (2), hence it is proved that x
T
y = tr(xy
T
).
Question # 07
Show that |A
T
| = |A|. Using this result, prove that for an orthogonal matrix A, |A|
2
= 1.
Solution-Proof (a)
A
t
is obtained from A by switching its rows and columns. Since we can compute the determinant by row
or column cofactor expansion and get the same answer, we can compute |A| by cofactor expansion along
the first row of A which is the same as cofactor expansion along the first column of A
t
. But the latter is
|A
t
|.
Now we can also proceed by induction on n.
For the base case n = 1, we get A = A
t
so that |A| = |A
1
|, as desired. Now for the inductive step assume
the result is true for n=k-1 and let A be a k k matrix. Consider
A = _
o
11
o
12
o
21
o
22

o
1k
o
2k

o
k1
o
k2 o
kk
_ so A
t
= _
o
11
o
21
o
12
o
22

o
k1
o
k2

o
1k
o
2k o
kk
_
Using cofactor expansion along the first column of A, we get as follows;
|A| = o
11
|A
11
| o
21
|A
21
| + + (1)
k+1
o
k1
|A
k1
|
Where A
]
is the matrix obtained from A by removing the ith row and jth column. Using cofactor expansion
along th fist row of A
t
we have
|A
t
| = o
11
|A
t
11
| o
21
|A
t
21
| + + (1)
k+1
o
k1
|A
t
k1
|
As (A
t
)
]
= (A
]
)
t
. Since A
]
is a (k 1) (k 1) matrix we can use the inductive hypothesis to see that
|A
t
|j
| = (A
j|
)
t
= |A
j|
|
Making thsi substitution into the above formula for |A
t
| evolves |A
t
| = |A|.
Solution-Proof (b):
EE 571 Linear Systems Home Work 1 Page 10 / 14

For an orthogonal matrix A,
= A
T
= A
-1

Post mutiplying the above equation by matrix A,
= A
T
A = A
-1
A = I
Now taking the determinant of both of the sides produces the following form;
|A
T
A| = |I|
= |A
T
A| = 1
= |A
T
||A| = 1
So,
|A
T
| = |A|
= |A||A| = 1
= |A|
2
= 1
Question # 08 (Problem 3.6 Chens Book)
Find basis of the range space and null spaces of the matrices.
A
1
= _
1

1
_ , A
2
= _
4 1 1
3 2
1 1
_ , A
3
= _
1 2
1

3 4
2 2
1
_
Solution (Range Space of A
1
):
A
1
= _
u 1 u
u u u
u u 1
_ , A
1
t
= _
u u u
1 u u
u u 1
_
The Jordan reduced echelon form of matrix (A
1
t
) is obtained as;
u = _
1 u u
u u u
u u 1
_
The original vectors corresponding to the all-non-zero rows are a basis for the range i.e;
I
1
= (u,u,u), I
2
= (u,u,1)
Solution (Null Space of A
1
):
A
1
= _
u 1 u
u u u
u u 1
_
The Jordan reduced echelon form is obtained as;
EE 571 Linear Systems Home Work 1 Page 11 / 14

u = _
1 u u
u u u
u u 1
_
Here x
2
is free variable and the rest x
1
, x
3
are basis variables. Now we shall solve the system ux = u
with x
2
= 1 to get the vector I
1
;
_
1 u u
u u u
u u 1
_ _
x
1
x
2
x
3
_ = _
u
u
u
_ = x
1
= u, x
3
= u
So I
1
= (u,1,u) is the basis of null space and the nullity of A
1
is 1.
Note the dimension of range is 2 and the dimension of nullity is 1.
Solution (Range Space of A
2
):
A
2
= _
4 1 1
S 2 u
1 1 u
_ , A
2
t
= _
4 S 1
1 2 1
1 u u
_
Now the Jordan reduced echelon form of matrix (A
2
t
) is obtained as under;
u = _
1 2 1
4 S 1
1 u u
_ R
1
R
2
= u = _
1 2 1
u S S
u 2 1
_ , R
2
4R
1
, R
3
+ R
1

= u = _
1 2 1
u 1
S
S
u 2 1
_ ,
1
S
R
2
= u =

1 u
1
S
u 1
S
S
u u
1
S

R
1
2R
2
, R
3
2R
2

= u =

1 u
1
S
u 1
S
S
u u 1

, SR
3
= u = _
1 u u
u 1 u
u u 1
_ , R
1
+
1
S
R
3
, R
2
+
S
S
R
3

The original vectors corresponding to the all-non-zero rows are a basis for the range i.e;
I
1
= (4,S,1), I
2
= (1,2,1), I
2
= (1,u,u)
Solution (Null Space of A
2
):
A
2
= _
4 1 1
S 2 u
1 1 u
_
Now the Jordan reduced echelon form of matrix (A
2
) is obtained as under;
u = _
1 1 1
S 2 u
1 1 u
_ R
1
R
2
= u = _
1 1 1
u S S
u 2 1
_ , R
2
SR
1
, R
3
R
1

EE 571 Linear Systems Home Work 1 Page 12 / 14

= u = _
1 1 1
u 1 1
u 2 1
_ , R
2
2R
3
= u = _
1 u u
u 1 1
u u 1
_ , R
1
+ R
2
, R
3
2R
2

= u = _
1 u u
u 1 u
u u 1
_ , 1R
3
, R
2
R
3

Since here x
1
= u, x
2
= u, x
3
= u. So it s basis is a null vectors.
Solution (Range Space of A
3
):
A
3
= _
1 2 S
u 1 2
u u u
4
2
1
_ , A
3
t
= _
1 u
2 1
S 2
u
u
u
4 2 1
_
Now the Jordan reduced echelon form of matrix (A
3
t
) is obtained as under;
u = _
1 u
u 1
u 2
u
u
u
u 2 1
_ , R
2
2R
1
, R
3
SR
1
, R
4
4R
1

= u = _
1 u
u 1
u u
u
u
u
u u 1
_ , 1R
2
, R
3
+ 2R
2
, R
4
2R
2

The original vectors corresponding to the all-non-zero rows are a basis for the range i.e;
I
1
= (1,u,u), I
2
= (2, 1,u), I
3
= (4,2,1)
Solution (Null Space of A
3
):
A
3
= _
1 2 S
u 1 2
u u u
4
2
1
_
Now the Jordan reduced echelon form of matrix (A
3
) is obtained as under;
u = _
1 u 1
u 1 2
u u u
8
2
1
_ , 1R
2
, R
1
+ 2R
2
= u = _
1 u 1
u 1 2
u u u
u
u
1
_ , R
1
8R
3
, R
2
+ 2R
3

Here x
3
is a free variable and the rest x
1
, x
2
, x
4
are basic variables.
Now we shall solve the system ux = u with x
3
= 1 for I
1
.
= _
1 u 1
u 1 2
u u u
u
u
1
_ _
x
1
x
2
x
3
x
4
_ = _
u
u
u
u
_
= x
1
x
3
= u , x
2
2x
3
= u, x
4
= u
= x
1
= 1, x
2
= 2, x
3
= 1, x
4
= u
EE 571 Linear Systems Home Work 1 Page 13 / 14

So I
1
= (1, 2, 1, u) is the basis of null space and nullity is 1.
Question # 09 (Problem 3.7 Chens Book)
Consider the linear Algebraic equation.
_
2 1
3 3
1 2
_ x = _
1

1
_ = y
It has 3 equations and two unknowns. Does a soluntion x exist in the equation? Is the solution unique?
Does a solution exist if y = |1 1 1]
c
?
Solution:
_
2 1
S S
1 2
_ x = _
1
u
1
_ (1)
Let
x = j
x
1
x
2
[
So (1) becomes as under;
_
2 1
S S
1 2
_ j
x
1
x
2
[ = _
1
u
1
_
Since the constant matrix has dimension 3*2 and the x matrix has dimension 2*1 so the multiplication of
constant matrix and variables matrix is possible which means solution is possible here.
= _
2x
1
x
2
Sx
1
+ Sx
2
x
1
+2x
2
_ = _
1
u
1
_
_
2x
1
x
2
= 1(2)
Sx
1
+ Sx
2
= u(S)
x
1
+ 2x
2
= 1(4)
_
Solving (2) and (3) gives x
1
= 1 & x
2
= 1. So the solution exists and is unique.
Now if y = |1 1 1]
c
= _
1
1
1
_ then
_
2 1
S S
1 2
_ j
x
1
x
2
[ = _
1
1
1
_ = _
2x
1
x
2
Sx
1
+Sx
2
x
1
+2x
2
_ = _
1
1
1
_
_
2x
1
x
2
= 1 (S)
Sx
1
+ Sx
2
= 1(6)
x
1
+ 2x
2
= 1(7)
_
Solving equations (5) and (7) gives x
1
= 1 & x
2
= 1, so the solution also exist when y = |1 1 1]
c
.
EE 571 Linear Systems Home Work 1 Page 14 / 14

Question # 10
Determine the four important subspaces of matrix A: R(A), N(A), R(A
T
), N(A
T
) where A = _
1
5 4
2 4
_.
Solution:
The dimension and basis of N(A):
A = _
1 u
S 4
2 4
_
Following the operations (R
2
SR
1
, R
3
2R
1
,
1
4
R
2
, R
3
4R
2
), the reduced echelon form is found to
be;
u = _
1 u
u 1
u u
_ (1)
Here x
1
, x
2
are basic variables and there is no free variable so the dimension of N(A) is zero.
The dimension and basis of R(A):
From (1) the corresponding non-zero columns of A are the basis of R(A) i.e; I
1
= (1, S, 2) and I
2
=
(u, 4, 4). So the dimension of R(A) = 2.
The dimension and basis of N(A
T
):
A
1
= j
1 S 2
u 4 4
[
Following the operations (
1
4
R
2
, R
1
SR
2
), the reduced echelon form is found to be;
u = j
1 u 9
u 1 1
[ (2)
Here x
3
is free variable and x
1
, x
2
are the basis variables. Now we will solve ux = u with x
3
= 1.
j
1 u 9
u 1 1
[ _
x
1
x
2
x
3
_ = j
u
u
[
By solving x
1
= 9 onJ x
2
= 1 are found. So I
1
= (9, 1, 1) is the basis of N(A
1
) and dimension of
N(A
1
) = 1
The dimension and basis of R(A
T
):
From (2) the corresponding non-zero columns of A are the basis of R(A
1
) i.e; I
1
= (1, u), I
2
= (S, 4). So
the dimension of R(A
1
) = 2.

Вам также может понравиться